0
$\begingroup$

This group theory problem has stumped me. I want to prove that if $G=(x)$ is a finite cyclic group that $(x^n) \cap (x^m) = (x^{\operatorname{lcm}(m,n)})$ for all integers $m$ and $n$, where $(x)$ is the group generated by $x$. Thoughts?

  • 4
    Can I asked why you posted the same question you posted an hour ago?2012-12-12
  • 1
    @user52714: please read the faq about how this site works: http://math.stackexchange.com/faq. Also please read: http://meta.math.stackexchange.com/questions/6714/can-i-ask-the-same-question-twice2012-12-12
  • 5
    @user52714 Note in particular, on the first of Thomas' links, the following: "Etiquette : Civility is required at all times; rudeness will not be tolerated."2012-12-12

3 Answers 3